extra practice questions exam 3

अब Quizwiz के साथ अपने होमवर्क और परीक्षाओं को एस करें!

A 25-year-old woman is seen because of numbness and pain in her fingers and hands on exposure to cold. When either her hands or feet are very cold, they turn white and then blue. These changes are characteristic of which of the following? (A) Buerger disease (thromboangiitis obliterans) (B) Raynaud disease (C) Wegener granulomatosis (D) Kawasaki disease (E) Takayasu arteritis

The answer is B. Raynaud disease is cold-induced vasospasm of arterioles and small arteries, most often involving the fingers and sometimes the hands and feet. Young, otherwise healthy women are most often affected.

A 20-year-old woman presents with depigmented white patches of skin on the face, neck, and hands. She has a past history of Graves disease. Which of the following is the most likely diagnosis? (A) Ocular albinism (B) Oculocutaneous albinism (C) Vitiligo (D) Freckle (E) Verruca vulgaris (common wart)

The answer is C. Vitiligo is an acquired loss of melanocytes in discrete areas of skin that appear as depigmented white patches. Vitiligo has no relationship to albinism. It is sometimes associated with autoimmune disorders, such as Graves disease, Addison disease, and antimelanocyte antibodies.

A 25-year-old woman presents with concern about a mole on her abdomen. The mole is small, round, symmetric, and darkcolored, and has sharply defined borders. It is diagnosed as a common mole. This lesion is properly termed a (A) nevocellular nevus. (B) blue nevus. (C) Spitz nevus. (D) dysplastic nevus. (E) lentigo maligna.

The answer is A. A melanocytic nevus (common mole) is a benign tumor. Nevus cells are derived from melanocytes and ordinarily occur in clusters or nests. A dysplastic nevus is an atypical, irregularly pigmented lesion with disorderly proliferation of melanocytes, dermal fibrosis, and lymphocytic infiltration. Dysplastic nevi may transform into malignant melanoma. Lentigo maligna is characterized by atypical melanocytes at the epidermal-dermal junction and is a precursor to lentigo maligna melanoma.

An 80-year-old woman, a retirement home resident, has multiple bouts of pneumonia caused by Streptococcus pneumoniae. In an attempt to prevent such infections, polyvalent vaccines directed at multiple serotypes of the organism have been administered but have not elicited long-acting immunity. Which of the following is the probable explanation for this phenomenon? (A) Memory T lymphocytes respond poorly to polysaccharide antigens. (B) S. pneumoniaeevades host immune response by forming capsular coatings composed of host proteins and recognized as "self" antigens. (C) The bacterial capsule binds C3b, facilitating activation of the alternative complement pathway, inducing complement-mediated lysis, and preventing immunization. (D) The capsular polysaccharides of S. pneumoniaehave limited hapten potential. (E) The surface carbohydrate capsule on the surface of the organism acts as an opsonin, facilitating phagocytosis by neutrophils, thus preventing immunization.

The answer is A. Antibody responses to the more than 80 differing carbohydrate capsular antigens of the various strains of S. pneumoniaeare generally T-cell-independent, and antibody formation is entirely B-cell-mediated. Because of this, memory cells are not formed, and long-lasting immunity is not achieved.

A 50-year-old man dies of a respiratory illness that had been characterized by dyspnea, cough, and wheezing expiration of many years' duration. Initially episodic, his "attacks" had increased in frequency and at the time of death had become continuous and intractable. At autopsy, which of the following is the most likely histologic finding in the lungs? (A) Bronchial smooth muscle hypertrophy with proliferation of eosinophils (B) Diffuse alveolar damage with leakage of protein-rich fluid into alveolar spaces (C) Dilation of air spaces with destruction of alveolar walls (D) Hyperplasia of bronchial mucussecreting submucosal glands (E) Permanent bronchial dilation caused by chronic infection, with bronchi filled with mucus and neutrophils

The answer is A. Bronchial asthma, or hyperreactive airway disease, is a type of COPD caused by narrowing of airways. Asthma manifests morphologically by bronchial smooth muscle hypertrophy, hyperplasia of bronchial submucosal glands and goblet cells, and airways plugged by mucus-containing Curschmann spirals (whorl-like accumulations of epithelial cells), eosinophils, and Charcot-Leyden crystals (crystalloids of eosinophil-derived proteins).

Yesterday, a 60-year-old man presented to the emergency department with dyspnea, diaphoresis, and crushing substernal chest pain that radiated to his neck and left arm. When asked to describe the pain, he put his fist to the center of his chest and stated that it felt "as if someone is squeezing my heart." An electrocardiogram demonstrated changes consistent with myocardial infarction, and serum troponin I levels were elevated. If the patient unexpectedly dies today, which of the following would almost certainly be found on histologic examination of the affected myocardium? (A) Coagulative necrosis with neutrophil infiltration (B) Fibrotic tissue replacing infarcted tissue (C) No histologic changes (D) Slight swelling of tissue and change of color (E) Young fibroblasts and new vessels growing into the infarcted tissue

The answer is A. By 24 hours well-developed microscopic changes of coagulative necrosis can be detected in infarcted tissue. There is loss of nuclei in cells and infiltration of neutrophils into tissue.

A 29-year-old man hospitalized for acquired immunodeficiency syndrome (AIDS) is found to have pulmonary tuberculosis. Which type of necrosis is found in the granulomatous lesions (clusters of modified macrophages) characteristic of this increasingly frequent complication of AIDS? (A) Caseous (B) Coagulative (C) Enzymatic (D) Fibrinoid (E) Liquefactive

The answer is A. Caseous necrosis occurs as part of granulomatous inflammation, typified by the lesions of tuberculosis.

A 53-year-old woman presents with dyspnea on exertion, orthopnea, paroxysmal nocturnal dyspnea, edema in the legs and feet, and fatigue. She has no history of angina, other signs of coronary artery disease, hypertension, or valvular disease. Echocardiography reveals cardiomegaly, with four-chamber hypertrophy and dilation. Which of the following is the most likely diagnosis? (A) Congestive or dilated cardiomyopathy (B) Hypertrophic cardiomyopathy (C) Myocarditis (D) Restrictive cardiomyopathy

The answer is A. Congestive or dilated cardiomyopathy is the most common form of cardiomyopathy. It is characterized by four-chamber hypertrophy and dilation as well as right- and left-sided severe heart failure. In some cases, congestive (dilated) cardiomyopathy may be associated with alcoholism, thiamine deficiency, or prior myocarditis.

A 4-year-old boy is seen after he suddenly develops a fever, abdominal pain and tenderness, hematuria, and palpable purpuric skin lesions on his buttocks and the extensor surfaces of the arms and legs. The most likely diagnosis is (A) Henoch-Schönlein purpura. (B) idiopathic (immune) thrombocytopenic purpura (ITP). (C) Kawasaki disease. (D) polyarteritis nodosa. (E) thrombotic thrombocytopenic purpura (TTP).

The answer is A. Henoch-Schönlein purpura is an IgA immune complex disease characterized by involvement of small vessels (venules, capillaries, arterioles) with multiple lesions, all about the same age, and is a form of hypersensitivity or leukocytoclastic vasculitis. The disorder may involve only the skin, presenting as palpable purpura, or it may involve a variety of other sites, including the glomeruli, gastrointestinal tract, lungs, or brain.

A 40-year-old woman in the 30th week of gestation presents to the emergency room because she has vaginal bleeding and lower abdominal pain. The uterus is tender to palpation and there are signs of fetal distress. Because of hematuria and rectal bleeding, disseminated intravascular coagulation (DIC) is suspected. Which of the following findings would be most supportive of the diagnosis of DIC? (A) Increased fibrin degradation products (B) Decreased activated partial thromboplastin time (APTT) (C) Decreased prothrombin time (PT) (D) Normal thrombin time (E) Thrombocytosis

The answer is A. In DIC, widespread thrombosis activates the fibrinolytic system, with degradation of both fibrin and fibrinogen; therefore, fibrin and fibrinogen degradation products are markedly increased. Since platelets are consumed, thrombocytopenia, not thrombocytosis, is an expected finding. In addition, the consumption of coagulation factors results in prolongation of the PT, a measure of the extrinsic pathway of coagulation; the APTT, a measure of the intrinsic pathway of coagulation; and the thrombin time, a measure of fibrinogen concentration. The clinical history strongly suggests premature separation of the placenta (abruptio placentae), a well-known cause of DIC.

A single nodule was resected from the peripheral portion of the right lower lobe of the lung of a 45-year-old woman. The microscopic findings were similar to those shown in the illustration. The diagnosis is (A) bronchioloalveolar carcinoma. (B) carcinoid. (C) mesothelioma. (D) small cell carcinoma. (E) squamous cell carcinoma.

The answer is A. In a typical well-differentiated bronchioloalveolar carcinoma, tumor cells line the walls of terminal air spaces, as shown in the illustration. When the tumor is localized to a single nodule, it is potentially curable by surgical resection.

An 80-year-old man presents with sharply demarcated, light brown, flat macules varying markedly in size. The lesions have the appearance of being "stuck on" or "pasted on," and they are particularly numerous on the trunk. Microscopically, sheets of small basaloid cells with some melanin production are seen. Keratin production occurs at the surface, and numerous small keratinfilled cysts are apparent. Which of the following is the likely diagnosis? (A) Seborrheic keratosis (B) Dermatofibroma (C) Keratoacanthoma (D) Actinic keratosis (E) Acanthosis nigricans

The answer is A. Seborrheic keratosis is an extremely common benign neoplasm occurring in older persons. This neoplasm is manifested by sharply demarcated, raised papules or plaques with a "pasted-on" appearance on the head, trunk, and extremities.

A 28-year-old woman is evaluated for possible thrombophilia since she has had two episodes of deep vein thrombosis, as well as two pregnancies that terminated in spontaneous abortion. The activated partial thromboplastin time is prolonged, and she has a positive VDRL screening test for syphilis. This combination of findings is most suggestive of (A) antiphospholipid antibody syndrome. (B) disseminated intravascular coagulation. (C) factor V Leiden. (D) methylene tetrahydrofolate reductase mutation. (e) prothrombin 20210A transition.

The answer is A. The combination of a prolonged activated partial thromboplastin time (APTT), a positive VDRL test for syphilis, recurrent thromboses (arterial or venous), and spontaneous abortion is highly suggestive of the antiphospholipid antibody syndrome. As the name implies, antibodies directed at phospholipids are a characteristic finding. Because of the prolonged APTT and frequent association with systemic lupus erythematosus (SLE), the antibody has been referred to as the "lupus anticoagulant," a misleading term because affected subjects have a thrombotic rather than hemorrhagic diathesis and not all subjects have SLE. The term primary antiphospholipid antibody syndrome is used when there is no evident underlying disease. It should be contrasted to secondary antiphospholipid antibody syndrome, in which the patient has a well-defined autoimmune disorder such as SLE.

A 24-year-old nonsmoker who does not consume alcohol is found to have both pulmonary emphysema and cirrhosis of the liver. A sister and several close relatives also have had similar findings. This history suggests that this patient's illness may be caused by deficiency of (A) α1-antitrypsin. (B) galactokinase. (C) glucose-6-phosphatase. (D) glucocerebrosidase. (E) phenylalanine hydroxylase.

The answer is A. The combination of emphysema and hepatic cirrhosis in a young person with a family history of similarly affected family members is strongly suggestive of homozygous α1-antitrypsin deficiency. In these instances the emphysema is usually panacinar in type.

During a laboratory exercise on coagulation testing, a 23-year-old medical student is found to have a prolonged bleeding time. She has had a long history of "easy bleeding," with frequent bleeding of the gums, epistaxis, cutaneous bleeding, and menorrhagia. Further testing revealed a deficiency of von Willebrand factor. Which of the following thrombogenic processes involving platelets is most directly impaired? (A) Adhesion (B) Conformational change with activation of phospholipid surface (C) Formation of fibrinogen bridges (D) Release reaction (e) Stabilization of platelet plug

The answer is A. von Willebrand factor is required for platelet adhesion to the subendothelium of damaged blood vessels. See further discussion of von Willebrand disease in Chapter 13.

A 1-year-old female infant is hospitalized for pneumonia. Bacterial cultures of the sputum have grown Pseudomonas aeruginosa. She has had two prior hospitalizations for severe respiratory infections. Her mother has noted that when she kisses her child, the child tastes "salty." The child has had weight loss that the mother attributes to frequent vomiting and diarrhea with bulky, foul-smelling fatty stools. The child is small for her age. Which of the following critical proteins is altered in this condition? (A) Cystic fibrosis transmembrane conductance regulator (B) Dystrophin (C) α-1,4-Glucosidase (D) α-L-Iduronidase (E) Lysyl hydroxylase

The answer is A. The diagnosis is cystic fibrosis, the most common lethal genetic disease in Caucasian populations. The disorder is due to a defect in the cystic fibrosis transmembrane conductance regulator protein, and about 70% of cases have a deletion of phenylalanine in position 508 (DF508 mutation). Affected patients often have multiple pulmonary infections and pancreatic insufficiency with steatorrhea and failure to thrive. Death is often due to respiratory failure secondary to repeated pulmonary infections, facilitated by the buildup of thick, tenacious mucus in the airways. Increased concentration of chloride in sweat and tears is characteristic, and the sweat test is an important diagnostic adjunct.

The myocardial lesions shown in the figure were observed at the autopsy examination of a pediatric patient who died after a short illness. During life, which of the following manifestations of his illness was most likely? (A) Chorea (B) Systemic embolization (C) Systemic lupus erythematosus (D) Unstable angina (E) Wasting diseases

The answer is A. The figure illustrates an Aschoff body, the characteristic lesion of rheumatic fever. This myocardial lesion is most often oval in shape and characterized by swollen, fragmented collagen and fibrinoid material and by characteristic large mesenchymal cells (Anitschkow myocytes) and multinucleated cells (Aschoff cells). Sydenham chorea is a major manifestation of rheumatic fever.

An 8-year-old child is evaluated by the pediatrician, who notes what appear to be 10 small café-au-lait spots on the child's torso. In addition, on close inspection of the eyes, the presence of Lisch nodules is noted. The patient is diagnosed with von Recklinghausen neurofibromatosis type 1. The protein that is mutated in this disorder normally (A) activates the GTPase activity of Ras. (B) cleaves cellular proteins during apoptosis. (C) functions as a regulator of the cell cycle. (D) promotes angiogenesis in the growing tumor mass. (E) promotes the cell to undergo apoptosis.

The answer is A. The normal function of NF-1 is to promote the intrinsic GTPase function of the Rasoncogene. When the Ras protein is bound to GTP, the growth-promoting function of the molecule is "ON." On hydrolysis of the GTP to GDP, Ras is converted to an inactive state. GAPs, such as NF-1, suppress cell growth by stimulating GTP hydrolysis. Patients with a mutation in NF-1 are susceptible to fibrosarcomas as a result of loss of function of this GAP. The molecule Bax is pro-apoptotic and antagonizes Bcl-2. Vascular endothelial growth factor promotes tumor angiogenesis. The protein p53 regulates the cell cycle if DNA damage is detected. Lastly, caspases function to cleave cellular proteins once apoptosis is triggered.

A 70-year-old man presents with a scaling, indurated, ulcerated nodule on the back of his left hand. He states that the nodule has been growing larger over time. The patient has had much direct sun exposure in the past. Which of the following is the most likely histologic finding in this patient's skin lesion? (A) Invasion of the dermis by sheets and islands of neoplastic epidermal cells, often with "keratin pearls" (B) Clusters of darkly staining basaloid cells, with a palisading arrangement of the nuclei of the cells at the periphery of the clusters (C) Malignant melanocytes with numerous mitotic figures (D) Abnormal proliferation of the connective tissue, with deranged arrangement of collagen fibers

The answer is A. This is a case of squamous cell carcinoma of the skin. Squamous cell carcinoma is a common skin tumor associated with excessive sun exposure. Clusters of darkly staining basaloid cells with a palisading arrangement of nuclei are characteristic of basal cell carcinoma. Malignant melanocytes are found in malignant melanoma. A keloid is a tumor-like scar resulting from abnormal proliferation of connective tissue with deranged arrangement of collagen fibers.

A 60-year-old man presents with angina. He has no past medical history of heart disease. On questioning, the patient reveals that he had repeated sexually transmitted diseases in the past, including a painless chancre (a hard, round sore) on his penis, for which he never sought medical attention. Rapid plasma reagin (RPR), Venereal Disease Research Laboratory (VDRL) slide test, and fluorescent treponemal antibody (FTA) serologic tests (indicative of syphilis infection) are positive. Echocardiography and computed tomography of the heart are performed. The history of untreated syphilis suggests that these tests will most likely detect which of the following abnormalities? (A) Aortic valvular insufficiency and linear calcification along the ascending aorta (B) Bicuspid aortic valve with aortic stenosis (C) Large valvular vegetations from bacterial endocarditis (D) Right-sided heart failure from the carcinoid syndrome (E) Small fibrin deposits on the mitral valve from nonbacterial thrombotic (marantic) endocarditis

The answer is A. This is a case of syphilitic (luetic) aortitis. In syphilitic aortitis, the elastica of the aorta undergoes calcification and is replaced by fibrous tissue, resulting in dilation of the ascending aorta and separation of the aortic valve commissures, with resultant aortic insufficiency. Thus, echocardiography and computed tomography of the heart reveal calcification in a linear pattern along the ascending aorta, calcification in the coronary arteries (leading to anginal symptoms), and aortic valvular insufficiency.

A 74-year-old woman presents to her primary care physician with malaise, proximal muscle pain, morning stiffness, and claudication of the jaw, along with occasional visual impairment. On examination, palpation along the lateral aspect of the head elicits tenderness. This is likely a result of (A) giant cell arteritis. (B) hypersensitivity vasculitis. (C) Kawasaki disease. (D) Takayasu arteritis. (E) Wegener granulomatosis.

The answer is A. This is the typical presentation of temporal, or giant cell, arteritis. Along with involvement of the branches of the carotid artery, such as the temporal or ophthalmic arteries, there is a close association with a complex of symptoms of proximal muscles termed polymyalgia rheumatica. Hypersensitivity vasculitis manifests with palpable purpura and is associated with drugs, food, or infectious agents. Takayasu arteritis involves the vessels of the aortic arch, whereas Kawasaki disease involves the coronary arteries. Wegener granulomatosis involves vessels of the respiratory tract and kidney.

A 36-year-old man dies during cardiac surgery. He had a history of long-standing rheumatic heart disease with mitral stenosis. At autopsy, the pathologist reports findings consistent with mitral stenosis and noted the presence of "heart failure cells." This finding results from (A) activation of the coagulation cascade. (B) chronic passive congestion of the lungs. (C) hypoxic myocardial injury. (D) myocardial hyperemia.

The answer is B. "Heart failure cells" are intra-alveolar hemosiderin-laden macrophages and are indicative of marked chronic passive congestion of the lung. Red cells leak from congested alveolar capillaries into the alveoli, where they are engulfed and degraded by macrophages.

A 70-year-old man seeks medical attention because of shortness of breath on minimal exertion. A posteroanterior chest radiograph reveals blunting of the right costophrenic sulcus interpreted as a rightsided pleural effusion. The aspirated fluid is straw colored and clear. The protein concentration is low, and the specific gravity is 1.011. Microscopic examination reveals an occasional mesothelial cell. Which of the following is the most likely cause of the effusion? (A) Decreased oncotic pressure (B) Left ventricular heart failure (C) Mesothelioma (D) Pneumonia (e) Tuberculosis

The answer is B. A clear, straw-colored fluid with low protein and low specific gravity is a transudate, and the term hydrothorax refers to the accumulation of a significant volume of transudate within the pleural cavities (to be detected by chest radiograph, about 200 to 400 mL of pleural fluid must be present). The most common cause of hydrothorax is cardiac failure, which may be either unilateral or bilateral (bilateral is more common). It is incumbent on the clinician to distinguish pleural transudates from exudates, because the causes of each are quite different.

A 45-year-old woman dies several days after a partial small bowel resection for repair of a volvulus. The surgery had apparently gone well, but shortly afterward she developed intractable fever, hypotension, multiorgan failure, and marked respiratory distress. Just prior to death, chest radiographs showed complete "whiteout" of both lungs. At autopsy, both lungs were found to have collapsed or distended alveoli, many of which were lined with fibrin-rich hyaline membranes. The cause of these pulmonary findings is best characterized as (A) aspiration. (B) diffuse alveolar damage. (C) generalized atelectasis. (D) lobar consolidation. (E) pneumothorax.

The answer is B. Alveolar hyaline membrane formation is a characteristic finding in ARDS. The common factor in ARDS is diffuse alveolar damage induced by a number of agents or conditions, one of which is septic shock. Other prominent causes include trauma, uremia, gastric aspiration, inhalation of chemical irritants, oxygen toxicity, Mycoplasmainfection, and the severe acute respiratory syndrome (SARS).

A 56-year-old physician who has had a recent episode of unstable angina is advised by his cardiologist to take one "baby aspirin" a day because of the antithrombotic effect of aspirin. What is the mechanism by which aspirin acts as an antithrombotic agent? (A) Acetylation and activation of both cyclooxygenase-1 (COX-1) and cyclooxygenase-2 (COX-2) (B) Acetylation and inhibition of both COX-1 and COX-2 (C) Selective inhibition of COX-1 (D) Selective inhibition of COX-2

The answer is B. Aspirin permanently acetylates the active site of cyclooxygenase (both COX-1 and COX-2), causing enzyme inhibition. This subsequently inhibits synthesis of the prothrombotic agent thromboxane A2. Thromboxane A2 causes activation and aggregation of platelets.

A 78-year-old man is found in his closed room unresponsive in bed after the first cold day of winter. There is a kerosene heater still on from the previous night. On attempts to arouse him, officers note the cherry hue of his lips, cheeks, and mucous membranes. The likely mechanism of his death was (A) accidental ingestion of ethylene glycol. (B) binding of carbon monoxide to hemoglobin. (C) hepatic necrosis with fatty change. (D) inhibition of hemoglobin production. (E) inhibition of incorporation of iron into hemoglobin.

The answer is B. Carbon monoxide (CO) binds to hemoglobin with approximately 200 times greater affinity than oxygen, preventing delivery of oxygen to tissues. In addition, CO is an inhibitor of cytochrome oxidase, impairing cellular respiration. Ethylene glycol (antifreeze) is associated with kidney damage and the formation of birefringent calcium oxalate crystals in the urinary tract. Hepatic necrosis and fatty change result from carbon tetrachloride poisoning. Lead inhibits a key enzyme in hemoglobin synthesis as well as iron incorporation in hemoglobin.

A 42-year-old man is seen because of a long history of slowly developing congestive heart failure. His blood pressure is normal. Coronary artery angiography reveals no vascular disease. No heart murmurs are heard. The white blood cell count, differential, and erythrocyte sedimentation rate are normal. The most likely diagnosis is (A) carcinoid heart disease. (B) cardiomyopathy. (C) coarctation of the aorta. (D) constrictive pericarditis. (E) myocardial infarction.

The answer is B. Cardiomyopathies are noninflammatory myocardial disorders that are not associated with coronary artery obstruction, hypertension, valvular disease, congenital heart disease, or infectious disease. They are most often characterized by otherwise unexplained ventricular dysfunction, such as cardiac failure, ventricular enlargement, or ventricular arrhythmias

A 56-year-old man is seen in the emergency room because of the acute onset of severe crushing precordial chest pain that began on the golf course 1 hour earlier and has persisted until the time of admission. The ECG reveals precordial QS waves and elevated ST segments. Although normal at admission, both the serum CK-MB and cTn-I are significantly elevated 12 hours later. These changes are related to which type of necrosis? (A) Caseous (B) Coagulative (C) Fat (D) Gangrenous (E) Liquefactive

The answer is B. Chest pain, ECG findings of QS waves with elevated ST segments, and elevated serum CK-MB and cTn-I are diagnostic of acute MI. MI is the prototype of coagulative necrosis

A 45-year-old man is referred because of a recent diagnosis of hereditary hemochromatosis. Which of the following is a correct statement about this disorder? (A) Damage to organs results from abnormal deposition of lead (B) It can progress to liver cirrhosis, diabetes mellitus, and skin pigmentation (C) Most cases are due to spontaneous mutations (D) Skin hyperpigmentation is due to bilirubin accumulation (E) The TIBC is characteristically increased

The answer is B. In advanced form, primary (hereditary) hemochromatosis is characterized by the triad of cirrhosis, diabetes, and hyperpigmentation, or so-called bronze diabetes. The disease is most often caused by a mutation in the Hfegene on chromosome 6 and is characteristically familial rather than sporadic. The manifestations of the disorder are the result of iron overload and deposition of hemosiderin in tissues such as the liver, pancreas, skin, joints, and pituitary. Laboratory abnormalities of note include increased serum iron and decreased TIBC. The skin hyperpigmentation is due largely to increases in melanin and to lesser accumulations of hemosiderin.

A 56-year-old man has severe chest pain, and angiography demonstrates acute occlusion of the circumflex branch of the left coronary artery. Blood flow is successfully restored by percutaneous transluminal coronary angioplasty (PTCA) with stenting. During the period of cellular hypoxic injury, which of the following intracellular changes is likely to have occurred? (A) Decreased calcium (B) Decreased pH (C) Decreased sodium (D) Increased activity of NaKpump (E) Increased adenosine triphosphate (ATP)

The answer is B. Intracellular pH is decreased in severe hypoxic cell injury. This change is caused by mitochondrial damage, which in turn results in decreased oxidative phosphorylation and diminished ATP synthesis. Decreased ATP stimulates glycolysis with lactate formation, thus resulting in decreased intracellular pH. Decreased ATP also diminishes the activity of the membrane-associated Na1K1 pump, allowing an influx of sodium and water. The final steps leading to cell death in severe hypoxic injury are associated with massive influx of extracellular calcium.

A 25-year-old woman has recently been diagnosed with a suspicious pigmented skin lesion and is waiting for the results of a biopsy. She is concerned because she had heard that such lesions vary, with some being more serious than others. Which of the following pigmented lesions is most likely to metastasize early? (A) Dysplastic nevus (B) Nodular melanoma (C) Pigmented nevus (D) Juvenile melanoma (E) Superficial spreading melanoma

The answer is B. Nodular melanoma tends to expand vertically rather than horizontally, a phenomenon associated with a more aggressive course and a greater likelihood of metastasis. Among the malignant melanomas, nodular melanoma has the poorest prognosis.

An 18-year-old young man with known cystic fibrosis presents to the physician with his third episode of kidney stones in the past year. In addition, he has begun to complain of difficulty seeing at night. Such changes can be attributed to a deficiency of which vitamin? (A) Pyridoxine (B) Vitamin A (C) Vitamin B1 (D) Vitamin B12 (E) Vitamin D

The answer is B. Patients with cystic fibrosis often have impaired exocrine pancreas function, resulting in a deficiency of fat-soluble vitamins. Of the fat-soluble vitamins, vitamin A deficiency results in night blindness, xerophthalmia, keratomalacia, and squamous metaplasia of the urinary tract, contributing to the formation of recurrent renal calculi.

A 56-year-old man with a history of stable angina was seen in the emergency room 1 hour following the onset of unrelenting substernal pain not relieved by nitroglycerin. An electrocardiogram (ECG) revealed deep Q waves across the precordium, ST segment elevations, and inverted T waves. Serum levels of creatinine kinase MB (CK-MB) and cardiac troponin I (cTn-I) were within the normal range. What is the best explanation for these findings? (A) Lactate dehydrogenase (LDH) should have been ordered rather than CK-MB and cTn-I. (B) The diagnosis is acute myocardial infarction (MI), and CK-MB and cTn-I were determined too early in the course of the disease. (C) The diagnosis is unstable angina rather than acute MI. (D) The findings are indicative of a dissecting aneurysm (dissecting hematoma) of the aorta. (E) CK-MB alone can be misleading, and more definitive information would have been expected from total CK determination.

The answer is B. Persistent chest pain unrelieved by nitroglycerin and the abnormal ECG findings are diagnostic of MI. It would be unusual to observe significant elevations of CK-MB and cTn-I as early as 1 hour following an MI. These markers rise in parallel and are weakly positive in about 6 hours and reach peak levels in about 10 to 15 hours following an MI. CK-MB returns to normal levels in 3 to 7 days, while cTn-I can remain elevated a week or longer.

A 67-year-old man, a heavy smoker, is seen because of dyspnea and cough. A chest X-ray reveals abnormal densities, and a computed tomography (CT) scan is suggestive of a neoplasm involving the pleura. A biopsy confirms the diagnosis of mesothelioma. Other than cigarette smoke, this finding suggests exposure to which toxin? (A) Aflatoxin B1 (B) Asbestos (C) Diethylstilbestrol (DES) (D) Ionizing radiation (E) β-Naphthylamine

The answer is B. Pleural and peritoneal mesotheliomas are associated with exposure to asbestos, and the apparent tumorigenic effect of asbestosis is markedly enhanced by cigarette smoking. Aflatoxin B1 is associated with hepatocellular carcinoma. Clear cell adenocarcinoma of the vagina has been a hazard to daughters exposed during intrauterine life to DES administered to their mothers to prevent spontaneous abortion. Ionizing radiation is associated with many cancers, including leukemias, breast cancer, and thyroid malignancies. β-naphthylamine and other aniline dyes are associated with transitional cell carcinoma of the bladder.

A 20-year-old woman presents with a skin rash. The rash is localized to the extensor surfaces of her elbows and knees, and is composed of multiple well-demarcated circular-to-oval-shaped plaques and papules covered by a silvery scale overlying reddened erythematous skin. The silvery scale can be scraped off, revealing inflamed skin underneath. She states that several family members have a similar rash. Which of the following is the most likely diagnosis? (A) Neurodermatitis (lichen simplex chronicus) (B) Psoriasis (C) Pemphigus vulgaris (D) Bullous pemphigoid (E) Dermatitis herpetiformis

The answer is B. Psoriasis is a chronic inflammatory skin disease characterized by erythematous plaques covered with a silvery scale. Histologic findings include epidermal proliferation with acanthosis, parakeratosis, and Munro abscesses (minute neutrophilic abscesses). Psoriasis is sometimes associated with a rheumatoid arthritis-like condition termed psoriatic arthritis.

A 27-year-old man who was badly burned in an industrial accident requires multiple skin grafting. While in the intensive care unit, he is found to have blood in his stools, and endoscopy confirms the presence of many small ulcers in his stomach. This complication is referred to as (A) aplastic anemia. (B) Curling ulcer. (C) Cushing ulcer. (D) Reye syndrome. (E) Stevens-Johnson syndrome.

The answer is B. Punctate ulcers associated with extensive burn injuries are known as Curling ulcers. A similar phenomenon occurs in patients with head trauma, in which the lesions are known as Cushing ulcers. Aplastic anemia can result from an idiosyncratic reaction in patients taking the antibiotic chloramphenicol. Likewise, sulfonamides can cause a necrotizing eruption around mucous membranes in some individuals. Reye syndrome is associated with extensive microvesicular fatty change of the liver in children taking aspirin during an acute viral illness. The Stevens-Johnson syndrome is characterized by erosions and crusts of the lips and oral mucosa as a component of an extensive form of erythema multiforme, a maculopapular, vesiculobullous eruption often related to drugs (such as sulfonamides), neoplasia, or connective tissue disorders.

A 38-year-old woman is found to have episodic headache, palpitation, and diaphoresis, along with severe hypertension. She is also found to have hyperglycemia, but diabetes mellitus has been ruled out. These findings suggest an endocrine tumor secreting which of the following hormones? (A) Antidiuretic hormone (ADH) (B) Catecholamines (C) Insulin (D) PTH (E) Prolactin

The answer is B. The association of episodic headache, palpitation, and diaphoresis, along with severe hypertension and hyperglycemia, is most suggestive of a catecholamine-secreting pheochromocytoma. Other nondiabetic endocrine disorders associated with hyperglycemia include Cushing syndrome, either pituitary or adrenal, with hypersecretion of corticotropin or cortisol; acromegaly, with hypersecretion of growth hormone; and hyperthyroidism, with hypersecretion of thyroxine.

A bedridden elderly patient experiences the sudden onset of pleuritic pain and hemoptysis. The underlying lesion that led to this complication was most likely located in which of the following sites? (A) Hepatic veins (B) Lower extremity veins (C) Pelvic veins (D) Portal vein (e) Pulmonary veins

The answer is B. The clinical description is characteristic of pulmonary infarction, which, in turn, most often results from thromboembolism originating from thrombosis in the lower extremity veins. Because venous thrombosis is associated with impaired blood flow, this condition is particularly characteristic of immobilization, which is often seen in elderly, debilitated, or chronically bedridden persons.

A 68-year-old woman presents with fever, chills, and cough productive of blood-tinged sputum. Fluid aspirated from the right pleural space would most likely (A) be clear and straw-colored in appearance. (B) contain large numbers of neutrophils. (C) have a glucose content somewhat higher than the serum glucose.(D) have a protein content of less than 1 g/dL. (E) have a specific gravity of 1.012.

The answer is B. The clinical history is strongly suggestive of bacterial pneumonia. Pleural fluid from this patient would typically be an exudate rather than a transudate, and would be expected to be cloudy and contain many neutrophils. The fluid would also demonstrate reduced glucose, increased protein, and increased specific gravity.

A 40-year-old woman presents with grayish pigmentation of the skin in a large region covering her entire posterior neck and axillae. The hyperpigmented areas started out as smaller macules but have now progressed to form palpable plaques. At times the hyperpigmented areas are pruritic. Which of the following is an important association of this skin lesion? (A) Hypercholesterolemia (B) Marker of visceral malignancy (C) Tends to recur after resection (D) Convulsions, mental retardation, and retinal detachment (E) Viral infection

The answer is B. The clinical presentation is illustrative of acanthosis nigricans, a cutaneous lesion that may be associated not only with diabetes and other endocrinopathies, but also with visceral malignancy, such as carcinoma of the stomach, lung, breast, or uterus. Other associations of malignancy include migratory venous thrombosis, which is also associated with visceral malignancies; clubbing of the fingers, which may be associated with a number of disorders, including carcinoma of the lung; and marantic endocarditis, which is associated with wasting diseases, such as widespread cancer.

This figure illustrates the microscopic appearance of the heart of a 56-year-old man who died after a 24-hour hospitalization for severe "crushing" chest pain complicated by hypotension and pulmonary edema. The type of necrosis shown is best described as (A) caseous. (B) coagulative. (C) fibrinoid. (D) gangrenous. (E) liquefactive.

The answer is B. The figure illustrates general preservation of myocardial architecture with some fragmentation, more intense cytoplasmic staining corresponding to increased cellular eosinophilia, and loss of nuclei, all of which are characteristics of coagulative necrosis.

A 3-year-old boy is brought by his parents to the emergency department because they are concerned that he has had a high fever for several days. On examination, the boy has conjunctival and oral erythema. He has palpable cervical lymphadenopathy and erythema of his palms and soles. What is a potential life-threatening complication of this disorder? (A) Aneurysm of the abdominal aorta (B) Aneurysm of the coronary arteries (C) Dissection of the thoracic aorta (D) Ischemia and gangrene of the extremities (E) Rupture of a berry aneurysm

The answer is B. The patient presents with mucocutaneous lymph node syndrome, or Kawasaki disease. It is often a self-limiting condition, although as many as 20% of patients develop an aneurysm of the coronary vessels. Dissection of the thoracic aorta is associated with hypertension. Rupture of a berry aneurysm can result in a subarachnoid hemorrhage. Aneurysm of the abdominal aorta is associated with atherosclerosis. Ischemia and gangrene can be a result of Buerger disease.

A 64-year-old woman presents with dependent peripheral edema in her ankles and feet. She has long-standing chronic obstructive lung disease and a long history of cigarette smoking. Further investigation reveals that she has cor pulmonale with right-sided heart failure. Which of the following is the most likely cause of the right-sided heart failure in this patient? (A) Constrictive pericarditis (B) Disease of the lungs or pulmonary vessels (C) Left-sided heart failure (D) Pulmonary infundibular or valvular stenosis (E) Systemic hypertension

The answer is B. The term cor pulmonale refers to right ventricular hypertrophy caused by pulmonary hypertension secondary to disorders of the lungs or pulmonary vessels. Other causes of right ventricular hypertrophy and failure, such as valvular disease, congenital defects, and left-sided heart failure, are precluded by this definition. Therefore, although in general, the most common cause of right-sided heart failure is left-sided heart failure, cor pulmonale with right-sided heart failure is due to an intrinsic disease originating in the lungs. Constrictive pericarditis can clinically mimic right-sided heart failure but is entirely unrelated to cor pulmonale.

A routine complete blood count performed on a 22-year-old medical student reveals an abnormality in the differential leukocyte count. She has been complaining of frequent sneezing and "watery" eyes during the past several weeks and reports that she frequently had such episodes in the spring and summer. Which of the following cell types is most likely to be increased? (A) Basophils (B) Eosinophils (c) Lymphocytes (d) Monocytes (e) Neutrophils

The answer is B. This type of reaction is primarily mediated by the release of histamine from tissue mast cells, and the associated cellular infiltrate and peripheral blood findings represent mobilization and increased numbers of eosinophils. The symptoms reported are those of seasonal rhinitis, better known as "hay fever," a manifestation of type I hypersensitivity (see Chapter 5).

A 78-year-old Navy veteran with a 600-pack-per-year history of cigarette smoking presents with cancer. During his military career, he was involved in fireproofing naval combat ships with asbestos insulation. Given his environmental exposure to both tobacco and asbestos, to which cancer do both of these carcinogens contribute? (A) Bladder cancer (B) Bronchogenic cancer (C) Cancer of the throat (D) Esophageal cancer (E) Mesothelioma

The answer is B. Tobacco contributes to the development of many cancers, including those of the bladder, lung, throat, and esophagus. Asbestos exposure carries a risk of lung cancer, as well as mesothelioma, a cancer of the pleura. Tobacco and asbestos function as cocarcinogens in the pathogenesis of lung cancer, with an approximately 50-fold greater risk of developing bronchogenic cancer of the lung than in those without such exposure.

An 8-year-old boy is referred to the dermatologist for numerous "suspicious" pigmented lesions on the face and neck. Further history reveals that the patient has had difficulty seeing out of his right eye; he is referred to the ophthalmologist, who diagnoses an ocular melanoma. Based on the patient's symptoms, the diagnosis of xeroderma pigmentosum is considered. This condition results from (A) aberrant expression of a receptor tyrosine kinase. (B) an inborn defect in DNA repair. (C) chemical carcinogenesis. (D) DNA viral infection. (E) retroviral infection.

The answer is B. Xeroderma pigmentosum is a hereditary DNA defect with a deficiency in the ability to repair ultraviolet (sunlight)-induced thymidine dimers. Faulty repair leads to increased sun sensitivity, with a predilection to develop skin lesions and skin cancers on exposed skin, as well as ocular melanomas. Aberrant expression of the gene for a receptor tyrosine kinase, such as the Her-2/neu gene product, can cause breast cancer. A retrovirus is responsible for the development of T-cell leukemia/lymphoma. The DNA virus human papillomavirus can cause cervical cancer.

A 23-year-old man presents with fever, weight loss, malaise, abdominal pain, and myalgias. Workup reveals that the patient has polyarteritis nodosa. Which of the following is associated with this form of vasculitis? (A) Arsenic (B) Chlamydia pneumonia (C) Hepatitis B virus (D) Human herpesvirus 8 (E) Human immunodeficiency virus

The answer is C. Although the significance remains unclear, 30% of patients with polyarteritis nodosa have serum antibodies to the hepatitis B virus. Arsenic is associated with the development of angiosarcoma. Chlamydia pneumonia has been implicated in the development of atherosclerosis. KSHV, HHV-8, causes Kaposi sarcoma in individuals infected with human immunodeficiency virus.

A 45-year-old man presents with abdominal pain and hypertension. On physical examination, he is found to have an abdominal mass. Further workup confirms the diagnosis of adult polycystic kidney disease. Which of the following vascular complications is associated with this condition? (A) Arteriovenous fistula (B) Atherosclerotic aneurysm (C) Berry aneurysm (D) Dissecting aneurysm (E) Luetic aneurysm

The answer is C. Berry aneurysms, which occur in 10% to 15% of patients with adult polycystic kidney disease, are small saccular lesions that develop at the site of congenital weakness of cerebral arteries, especially those of the circle of Willis. Rupture of these aneurysms is the most common cause of subarachnoid hemorrhage. Arteriovenous fistulas are often secondary to trauma. Dissecting aneurysm is associated with hypertension or with diseases affecting the vascular media, most notably Marfan syndrome. Syphilitic (luetic) aneurysm is associated with tertiary syphilis.

The chest radiograph of a 23-year-old medical student reveals a calcified cavitary pulmonary lesion. The tuberculin test is positive, but sputum smears and cultures are negative for Mycobacterium tuberculosis. A presumptive diagnosis of secondary tuberculosis is made. If further studies, including a biopsy, were performed, which of the following findings would justify the diagnosis of secondary tuberculosis, as contrasted to primary tuberculosis? (A) Calcification (B) Caseating granulomas (C) Cavitation (D) Langhans giant cells (E) Positive tuberculin test result

The answer is C. Cavitation occurs only in secondary tuberculosis. Both primary and secondary tuberculosis are characterized by caseating granulomas, often with Langhans giant cells, which heal by scarring and calcification. The skin test result for tuberculin sensitivity is positive in both forms.

A 60-year-old man, a heavy smoker, presents for advice to stop smoking. On physical examination, he is thin and has a ruddy complexion. He has a productive cough and a barrel-shaped chest. He sits leaning forward with his lips pursed to facilitate his breathing. A diagnosis of emphysema is made. Which of the following is the most likely histologic finding in the lungs? (A) Bronchial smooth muscle hypertrophy with proliferation of eosinophils (B) Diffuse alveolar damage with leakage of protein-rich fluid into alveolar spaces (C) Dilation of air spaces with destruction of alveolar walls (D) Hyperplasia of bronchial mucus-secreting submucosal glands (E) Permanent bronchial dilation caused by chronic infection, with bronchi filled with mucus and neutrophils

The answer is C. Emphysema is an example of COPD. Due to the destruction of alveolar walls, a lack of elastic recoil causes air to become trapped in alveoli, and, thus, airflow obstruction occurs on expiration. In COPD, FEV1 is decreased, whereas FVC is normal or increased; therefore, patients with COPD have a decreased FEV1:FVC ratio.

A 60-year-old man presents with the new onset of dyspnea, chest pain, cough, and weight loss. In the past, he had worked in construction, installing insulation in buildings. Chest radiograph shows a rightsided pleural effusion with marked pleural thickening and mass formation. A malignant neoplasm is demonstrated by biopsy. What is the most likely diagnosis? (A) Small cell carcinoma (B) Squamous cell carcinoma (C) Mesothelioma (D) Adenocarcinoma (E) Carcinoid tumor

The answer is C. Exposure to asbestos (common in construction workers, shipyard workers, or people who have worked with insulation or fire safety materials) markedly predisposes one to mesothelioma of the pleura or peritoneum and is also closely linked to primary lung carcinoma (especially in smokers).

A 56-year-old man collapsed at work and died 20 minutes later in the emergency room while blood was being drawn. The patient's history revealed an episode of prolonged chest discomfort 3 months earlier. Which of the following is least likely? (A) Death from arrhythmia (B) Fibrotic scar in the ventricular septum (C) Loss of myocardial striations and beginning infiltration with neutrophils (D) Normal values for serum creatine kinase, troponin I, and myoglobin (E) Severe atherosclerotic narrowing of the anterior descending branch of the left coronary artery with overlying thrombus formation

The answer is C. In the first several hours after MI, the most common cause of death is arrhythmia. Although evidence of acute coronary artery obstruction may be found, morphologic myocardial changes and serum myocardial marker protein elevations are most often delayed for several hours. A myocardial fibrotic scar is evidence of an old prior MI.

A 25-year-old man presents with a progressive illness of several days' duration characterized by nonproductive cough, fever, and malaise. A lateral view chest radiograph reveals platelike atelectasis. Elevated titers of cold agglutinins are detected. Which of the following is the most likely type of pneumonia in this patient? (A) Bacterial pneumonia, most likely caused by S. pneumoniae (B) Hospital-acquired pneumonia, most likely caused by P. aeruginosa (C) Interstitial pneumonia, most likely caused by M. pneumoniae (D) P. jiroveci(carinii) pneumonia, most likely related to an immunocompromised state (E) Viral pneumonia, most likely caused by influenza virus

The answer is C. Interstitial (primary atypical) pneumonia is most commonly caused by M. pneumoniaeor viruses. Interstitial pneumonia is characterized by diffuse, patchy inflammation localized to the interstitial areas of alveolar walls, with no exudate in alveolar spaces, and intra-alveolar hyaline membranes. M. pneumoniaeinfection is associated with the presence of cold agglutinins, which are IgM antibodies that react nonspecifically with all human red blood cells. P. jiroveci (carinii)pneumonia is the most common opportunistic infection in patients with acquired immunodeficiency syndrome or other immunodeficiency disorders. Viral pneumonias are the most common type of pneumonia in childhood, caused most commonly by the influenza virus.

A 55-year-old man presents with a large, black-colored, asymmetric skin lesion with ill-defined borders on his back. He reports a family history of malignant melanoma. Which of the following clinical variants of malignant melanoma has the poorest prognosis? (A) Lentigo maligna melanoma (B) Superficial spreading melanoma (C) Nodular melanoma (D) Acral-lentiginous melanoma

The answer is C. Malignant melanoma arises from melanocytes or nevus cells, is most often associated with excessive sun exposure, and is most common in fair-skinned persons. Of the clinical variants of malignant melanoma, nodular melanoma has the worst prognosis. Malignant melanomas have a better prognosis when characterized by a long period of radial (superficial) growth, as opposed to early vertical growth (as in nodular melanoma).

A female infant is born prematurely at 28 weeks' gestation. Shortly after birth, she shows signs of dyspnea, cyanosis, and tachypnea. She is placed on a ventilator for assisted breathing, and a diagnosis of neonatal respiratory distress syndrome (hyaline membrane disease) is made. Which of the following is the cause of this syndrome? (A) Bronchopulmonary dysplasia (B) Intraventricular brain hemorrhage (C) Lack of fetal pulmonary maturity and deficiency of surfactant( D) Necrotizing enterocolitis (E) Patent ductus arteriosus

The answer is C. Neonatal respiratory distress syndrome (hyaline membrane disease) is the most common cause of respiratory failure in newborns and results from a deficiency of surfactant and immature development of the lungs. Surfactant reduces surface tension within the lung, facilitating expansion by inspiration and thus preventing atelectasis during expiration. The classically referenced indicator of fetal pulmonary maturity is a lecithin:sphingomyelin ratio of approximately 2:1 in the amniotic fluid, although techniques like lamellar body counts and the fluorescence polarization assay are now more commonly used to evaluate fetal lung maturity. Predisposing factors include prematurity, maternal diabetes mellitus, and birth by cesarean section. Known complications of this condition include bronchopulmonary dysplasia, patent ductus arteriosus, intraventricular brain hemorrhage, and necrotizing enterocolitis.

A 50-year-old woman has been immobilized in bed for several days after a motor vehicle accident. She had been improving, but this morning she suffered the sudden onset of pleuritic chest pain, hemoptysis, tachypnea, tachycardia, and dyspnea. What is the likely basis of this set of findings? (A) Arterial thrombus originating in pulmonary blood vessels (B) Arterial thrombus originating in the lower extremities with migration to pulmonary veins (C) Deep venous thrombus of the lower extremities with embolization to branches of the pulmonary artery (D) Mural thrombus originating in the left heart with migration to pulmonary blood vessels (E) Venous thrombus originating in pulmonary blood vessels

The answer is C. Pulmonary embolism most often originates from venous thrombosis in the lower extremities or pelvis. An embolus migrates through the venous circulation to the right heart and gets trapped in branches of the pulmonary artery. Pulmonary embolism occurs in clinical settings of venous stasis, such as primary venous disease, congestive heart failure, prolonged bed rest or immobilization, or prolonged sitting while traveling.

A 28-year-old woman is found to have pulmonary sarcoidosis. Flow cytometric analysis of T cells isolated from the alveoli and lung interstitium reveals the presence of large numbers of T helper (TH1) cells. These cells are known to secrete which of the following substances? (A) Complement component C5A (B) Elastase and lysyl-hydroxylase (C) Interleukin-2 (IL-2) and interferon-γ (D) IL-8 and transforming growth factor-β (E) Leukotrienes C4and C5

The answer is C. T helper (TH1) cells secrete IL-2 and interferon-γ, which in turn facilitate T-cell expansion and macrophage activation.

The accompanying figure is representative of the findings in a hilar lymph node from a 54-year-old man who sought medical care for low-grade fever, anorexia, fatigue, night sweats, weight loss, and persistent cough with bouts of hemoptysis. A chest x-ray had revealed a right apical infiltrate with beginning cavitation, and examination of the sputum had revealed acid-fast bacilli. This condition is typified by a form of inflammation that invariablyincludes which of the following? (A) A morphologically identifiable etiologic agent (B) Caseous necrosis (c) Clusters of epithelioid cells (d) Multinucleated giant cells (e) Prominent granulation tissue

The answer is C. The clinical description and the figure are both typical of advanced secondary tuberculosis. Although this disorder is now relatively uncommon, its incidence is increasing, especially in association with immunodeficiency. Tuberculosis is a classic cause of granulomatous inflammation, which is characterized by the presence of "granulomas," which by definition consist of clusters of modified macrophages referred to as epithelioid cells. Additional features such as caseous necrosis, giant cell formation, and identifiable etiologic agents may or may not be present and are not invariable features of this form of inflammation. Granulation tissue is a feature of early repair and is totally unrelated to granulomatous inflammation.

This segment of the arterial intima of a coronary vessel is observed at autopsy in a 56-year old man who died suddenly on rising in the morning. Which of the following abnormalities is considered a major risk factor for the development of this lesion? (A) Congenital vascular muscle weakness (B) Cystic medial necrosis (C) Hypercholesterolemia (D) Medial calcification (E) Syphilis

The answer is C. The figure shows a segment of a large atheromatous plaque. The clinical scenario strongly suggests that this affected vessel is a coronary artery. Cleft-like spaces indicating the presence of cholesterol crystals are very prominent and help in the identification of the lesion. The incidence of atherosclerosis is strongly associated with hypercholesterolemia. Renal disease is the most frequent cause of secondary hypertension. Endocrine disorders, such as pheochromocytoma, Conn syndrome, and acromegaly, represent the next most common cause. Coarctation of the aorta is a frequent cause of hypertension limited to the upper extremities.

A 32-year-old woman returns to her physician for follow-up of hypertension that has been poorly controlled in spite of numerous antihypertensive medications. It is decided to evaluate the patient for possible "secondary" hypertension. Which of the following is a well-known cause of secondary hypertension? (A) Ethnicity (B) Obesity (C) Renal artery stenosis (D) Smoking (E) Stress

The answer is C. The great majority of cases of hypertension are classified as essential hypertension. Essential hypertension results from the interactions of predisposing determinants and exogenous factors, including family history, ethnicity, stress, obesity, sodium intake, smoking, and physical activity. A small minority of cases of hypertension are due to secondary causes. Unilateral renal artery stenosis is a secondary cause that is typically correctable by surgery.

Three days after being admitted to the hospital for treatment of a gunshot wound, a 29-year-old man suffered the onset of acute respiratory distress, and diffuse bilateral infiltrates were seen in both lung fields on chest X-ray. A lung biopsy revealed the presence of intra-alveolar edema, along with hyaline membrane formation. These findings are indicative of (A) bacterial pneumonia. (B) viral pneumonia. (C) diffuse alveolar damage. (D) pulmonary hypertension. (E) left-sided heart failure.

The answer is C. The history is consistent with ARDS. ARDS is a cause of severe life threatening respiratory insufficiency and may be caused by a variety of etiologic agents, among them severe trauma, such as a gunshot wound. The common feature, regardless of etiology, is diffuse alveolar damage.

A 60-year-old man with pancreatic cancer dies following a terminal episode that began 3 days earlier, characterized by generalized bleeding with oozing from intravenous infusion sites, widespread petechial and ecchymotic cutaneous bleeding, and intractable epistaxis. At autopsy, a glomerular change similar to that shown in the illustration was demonstrated (the lesion is stained with a special stain that stains fibrin purple). Prior to death, which of the following laboratory measures of blood coagulation would have shown a decrease in the reported value? (A) Activated partial thromboplastin time (B) Fibrin and fibrinogen degradation products (C) Platelet count (D) Prothrombin time (E) Thrombin time

The answer is C. The illustration demonstrates thrombotic obliteration of glomerular capillary loops and is typical of DIC. In DIC—sometimes termed consumption coagulopathy—coagulation factors, fibrinogen, and platelets are depleted by the widespread thrombotic process. Thus the coagulation assays (APPT, PT, and thrombin time) are prolonged and the platelet count is decreased. Increased fibrin and fibrinogen degradation products are sensitive indicators of DIC.

A 23-year-old man presents with radiographic evidence of bilateral hilar lymphadenopathy and interstitial lung disease. A lung biopsy gives findings similar to those shown in the figure. A major characteristic of this disorder is (A) a positive test for tuberculin. (B) highest incidence in persons of Asian lineage. (C) hypercalcemia. (D) hypogammaglobulinemia. (E) involvement restricted to the lung.

The answer is C. The illustration shows noncaseating granulomas and giant cells, which, in the clinical setting described, are diagnostic of sarcoidosis. A frequent abnormal laboratory finding is polyclonal hypergammaglobulinemia along with hypercalcemia. Anergy to tuberculin is often demonstrable. The disorder is much more common in persons of African lineage. Patients most often present with lung findings and hilar lymphadenopathy, but any organ system can be involved

A scaling, ulcerated lesion develops on the forearm of a 45-year-old fisherman. Excisional biopsy is performed, and the histologic appearance is similar to that shown in the figure. Which of the following is most applicable to this lesion? (A) Distal metastases almost always occur (B) Indicative of an underlying visceral malignancy (C) Predilection for sun-exposed areas (D) Uncommon skin tumor

The answer is C. The lesion shown in the figure is a well-differentiated squamous cell carcinoma demonstrating sheets of neoplastic epidermal cells with keratin "pearls," a very common skin tumor. There is a marked predilection for sun-exposed areas, and most lesions occur on the lower part of the face or the back of the hands. Metastasis occurs in fewer than 5% of cases, because most of these lesions are discovered early and are cured by ablative therapy.

A 9-year-old girl is diagnosed with acute rheumatic fever. Instead of recovering as expected, her condition worsens, and she dies. Which of the following is the most likely cause of death? (A) Central nervous system involvement (B) Endocarditis (C) Myocarditis (D) Pericarditis (E) Streptococcal sepsis

The answer is C. The most common cause of death that occurs during acute rheumatic fever is cardiac failure secondary to myocarditis.

A 4-year-old girl has had multiple hospitalizations for pneumonia. Additionally, she is small for her age and has had symptoms of fat malabsorption. Her father refers to her as his "little potato chip" because she tastes salty when he kisses her. Chest radiograph demonstrates pulmonary infiltrates indicative of pneumonia. Which of the following is the most likely pathogen causing pulmonary infection in this patient? (A) Legionella pneumophila (B) Haemophilus influenzae (C) Pseudomonas aeruginosa (D) Staphylococcus aureus (E) Streptococcus pneumoniae

The answer is C. This patient has cystic fibrosis. Cystic fibrosis is an autosomal recessive disease caused by mutations in the CFTR gene, which encodes a protein that functions as a chloride channel. Pseudomonas aeruginosais the most likely pathogen causing chronic pulmonary infection and pulmonary failure, and is the leading cause of death in patients with cystic fibrosis. P. aeruginosais also (after S. aureusand E. coli) the leading cause of nosocomial (hospital-acquired) infections and a frequent cause of death from burns.

A 10-year-old girl presents with an orange-red, dome-shaped papule on her right leg. Her parents state that the papule has grown from a tiny dot to over a centimeter in length in just a few months. Following biopsy, the dermatologist reassures the parents, stating that although the papule superficially resembles malignant melanoma, it is actually a benign lesion without malignant potential. Which of the following is the likely diagnosis? (A) Acanthosis nigricans (B) Actinic keratosis (C) Dysplastic nevus (D) Spitz nevus (E) Xeroderma pigmentosum

The answer is D. A Spitz nevus is a pediatric skin neoplasm that can be easily confused with malignant melanoma based on gross and microscopic morphology. It was formerly called "juvenile melanoma" however this name has fallen out of favor because it evokes unnecessary alarm, given the typically benign course of these tumors. Acanthosis nigricans is sometimes an indicator of visceral malignancy. Actinic keratosis is a premalignant epidermal lesion. Dysplastic nevus may transform into malignant melanoma. Xeroderma pigmentosum is associated with a markedly increased incidence of skin cancer caused by failure of DNA repair.

A 17-year-old African-American girl recently had her ears pierced for the first time. She now presents to her primary care provider with the complaint of a large "tumor-like" growth in the immediate site of one of the piercings. This lesion is likely a (A) benign fibrous histiocytoma. (B) dermatofibrosarcoma protuberans. (C) fibroepithelial polyp. (D) keloid. (E) xanthoma.

The answer is D. A keloid is a result of excessive production of collagenous fibrous tissue and is characterized by a tumorlike scar consisting of dense bundles of structurally abnormal collagen. Keloids have a marked tendency to recur after resection. Propensity to keloid formation is markedly increased in persons of African lineage.

An autopsy is performed on a 35- year-old African-American man who died after a brief illness characterized by papilledema, severe hypertension, left ventricular hypertrophy and failure, and renal dysfunction. The most likely findings in the kidney are (A) finely granular renal surface and hyaline arteriolosclerosis of afferent arterioles. (B) swollen, hypercellular, "bloodless" glomeruli. (C) nodular mesangial accumulations of basement membrane-like material and hyaline arteriolosclerosis of afferent and efferent arterioles. (D) surface covered with multiple petechial hemorrhages, hyperplastic arteriolosclerosis, and necrotizing glomerulitis. (E) swollen, pale kidneys and marked accumulation of lipid in convoluted tubules.

The answer is D. A rapidly fatal course with severe hypertension, left ventricular hypertrophy and failure, papilledema, and renal dysfunction is characteristic of malignant hypertension. This syndrome is most frequently seen in relatively young African-American men. The defining renal arteriolar lesion, malignant nephrosclerosis (hyperplastic arteriolosclerosis, fibrinoid necrosis, necrotizing arteriolitis), and the associated necrotizing glomerular lesion result in capillary rupture and the consequent "flea-bitten" appearance of the surfaces of the kidneys due to petechial hemorrhages.

A 60-year-old woman with a heavy smoking history presents with chronic productive cough that has been present for three consecutive months over the past two consecutive years. On physical examination, her skin has a bluish tinge, and she is overweight. The patient is diagnosed with chronic bronchitis. Which of the following is the most likely histologic finding in this patient's lungs? (A) Bronchial smooth muscle hypertrophy with proliferation of eosinophils (B) Diffuse alveolar damage with leakage of protein-rich fluid into alveolar spaces (C) Dilation of air spaces with destruction of alveolar walls (D) Hyperplasia of bronchial mucussecreting submucosal glands (E) Permanent bronchial dilation caused by chronic infection, with bronchi filled with mucus and neutrophils

The answer is D. Chronic bronchitis is an example of COPD. The pathologic hallmark of chronic bronchitis is marked hyperplasia of bronchial submucosal glands and bronchial smooth muscle hypertrophy, which can be quantified by the Reid index, a ratio of glandular layer thickness to bronchial wall thickness.

A 67-year-old man, a two-pack-a-day smoker since age 18, has had a productive cough over the past 20 years. Although continuous through the years, there have been episodic exacerbations of these symptoms, which have worsened during the past 4 or 5 years, lasting 3 or 4 months at a time. Arterial pO2 is decreased and pCO2 is increased. Total lung capacity measurements are normal. These findings are most suggestive of which of the following pulmonary disorders? (A) Adult respiratory distress syndrome (ARDS) (B) Bronchial asthma (C) Lung carcinoma (D) Chronic bronchitis (E) Panacinar emphysema

The answer is D. Chronic bronchitis, which is clearly linked to cigarette smoking, is defined as productive cough occurring during at least 3 consecutive months over at least 2 consecutive years.

A 25-year-old woman has cauliflowershaped perineal lesions that are diagnosed as condyloma acuminatum. The etiologic agent is (A) HSV. (B) Treponema pallidum. (C) Haemophilus ducreyi. (D) HPV. (E) Candida albicans.

The answer is D. Condyloma acuminatum is a nonmalignant neoplastic-like papillomatous condition that affects epithelium of the perineal, vulval, perianal, and vaginal regions. It is caused by HPV (most often HPV 6 and 11). Genital herpes is usually caused by HSV type 2. T. pallidumis a spirochete and the etiologic agent of syphilis. H. ducreyi causes chancroid, a painful, erosive, chancre-like lesion. Candidais not a sexually transmitted organism and is often considered normal vaginal flora, although it is an important cause of vulvovaginitis.

A 25-year-old man presents with a skin rash of "target" lesions on the palms of the hands, the soles of the feet, and the arms and legs. The rash is composed of macules, papules, and vesicles. The dermatologist makes a diagnosis of erythema multiforme. Which of the following is the probable etiology of this condition? (A) Physical scratching of the skin (B) IgG autoantibodies directed against the epidermal intercellular cement substance (C) IgA antibody deposits localized to the tips of dermal papillae (D) Hypersensitivity to multiple concomitant infectious agents or drugs (E) Chemical exposure

The answer is D. Erythema multiforme manifests as a variegated group of lesions: macules, papules, and vesicles. The most characteristic of these is the "target" lesion. The disorder is thought to be due to hypersensitivity, often to coexistent stimuli, such as infectious agents or drugs, or to a concomitant connective tissue disorder or an associated malignancy.

A 4-year-old girl is brought by her mother to the emergency department after the girl was "placed in a hot bath." The patient appears to have extensive blistering of the thighs and buttocks, with slight damage to the underlying dermis. Her burns are best described as (A) first-degree burns. (B) fourth-degree burns. (C) full-thickness burns. (D) second-degree burns. (E) third-degree burns.

The answer is D. First- and second-degree burns are both partial-thickness burns. Because the findings in this patient reveal both epithelial and dermal involvement, the burns are, by definition, second-degree in type. Both third- and fourth-degree burns are classified as full-thickness burns; they entail total destruction of dermis and epidermis along with underlying skin appendages that normally serve as a source of cells for regeneration. As such, these wounds require skin grafting.

A 60-year-old man with unstable angina (a form of acute coronary syndrome) is treated with an intravenously administered glycoprotein IIb-IIIa inhibitor. The mechanism of action of this agent is the ability to (A) dilate coronary arteries. (B) inhibit atherogenesis. (C) inhibit platelet adhesion. (D) inhibit platelet aggregation. (e) lyse thrombi.

The answer is D. Glycoprotein IIb-IIIa inhibitors prevent the action of the corresponding platelet surface receptor glycoprotein complex, which is required for formation of fibrinogen bridges between adjacent platelets.

A 56-year-old man recovered from a myocardial infarction after his myocardium was entirely "saved" by immediate thrombolytic therapy. If it had been possible to examine microscopic sections of his heart during his ischemic episode, which of the following would be the most likely cellular change to be found? (A) Karyolysis (B) Karyorrhexis (C) Pyknosis (D) Swelling of the endoplasmic reticulum

The answer is D. If infarction is averted by immediate thrombolytic therapy, indicators of necrosis, such as karyorrhexis, pyknosis, and karyolysis, which represent irreversible changes, would not be expected. Swelling of the endoplasmic reticulum from increased cell water, one of the earliest ultrastructural changes observed in injured cells, is reversible and would be expected.

A 16-year-old boy is referred to a dermatologist by an emergency room physician. The patient has had an intractable, severe, itching, burning, pruritic rash of the hands and lower extremities that has lasted several hours. Earlier in the day he had hiked through a wooded area filled with brush but was unaware of any direct contact with plants or other possible irritants. A skin biopsy revealed many infiltrating T cells and macrophages, suggesting an immune hypersensitivity reaction. Which type of reaction is most likely? (A) Type I (immediate or anaphylactic) hypersensitivity (B) Type II (antibody-mediated or cytotoxic) hypersensitivity (C) Type III (immune complex) hypersensitivity (D) Type IV (cell-mediated) hypersensitivity

The answer is D. In spite of the somewhat atypical presentation, the histologic findings are typical of contact dermatitis, which is a classic example of T cell-mediated (type IV) hypersensitivity.

A 3-year-old boy presents with cyanosis and shortness of breath that develops when he plays with friends. According to his mother, the boy was born cyanotic. The boy is very small and short for his age, and he squats on the floor next to his mother. Chest radiography reveals a boot-shaped heart, normal heart size, and a right aortic arch. Echocardiography reveals a large ventricular septal defect with an overriding aorta, pulmonary stenosis, and right ventricular hypertrophy. Which of the following is the most likely diagnosis? (A) Coarctation of the aorta (B) Patent ductus arteriosus (C) Rheumatic heart disease (D) Tetralogy of Fallot (E) Transposition of the great vessels

The answer is D. In the tetralogy of Fallot, the characteristic lesions include ventricular septal defect, overriding aorta, pulmonary valve stenosis, and right ventricular hypertrophy. The pulmonary stenosis and overriding aorta cause increased right ventricular pressure and lead to right-to-left shunting. Cyanosis, which occurs when the arterial concentration of reduced hemoglobin exceeds 5 mg/mL, is seen with a right-to-left shunt, in which venous blood gains direct access to the arterial circulation. In contrast, patent ductus arteriosus, atrial septal defect, and ventricular septal defect are associated with left-to-right blood flow.

A 19-year-old young woman who emigrated from Taiwan 8 years ago presents with fever, malaise, myalgias, and arthritis and "coldness" in her upper extremities. She has a weak radial pulse bilaterally, and a magnetic resonance angiogram demonstrates nearly 75% stenosis of the main arteries originating from the aorta. She likely has which of the following rheumatologic conditions? (A) Buerger disease (B) Kawasaki disease (C) Raynaud disease (D) Takayasu arteritis (E) Temporal arteritis

The answer is D. Inflammation and stenosis of branches of the aortic arch is known as Takayasu arteritis, or "pulseless disease." It most commonly occurs in young Asian females. Buerger disease usually affects young Jewish males and involves the arteries of the extremities. The disease is exacerbated by smoking and can lead to gangrene of the extremities. Kawasaki disease affects the branches of the coronary arteries. Raynaud disease is due to vasospasm of small vessels of the fingers and toes, leading to cyanosis and pallor of the affected tissues. Temporal arteritis is usually encountered in older patients and affects the branches of the carotid artery, most commonly the temporal artery.

A 19-year-old college sophomore is referred by his ophthalmologist because of the finding of ectopia lentis (dislocation of the lens), which has resulted in visual difficulties that have interfered with his performance on the varsity basketball team. The patient is very tall, with long limbs and long, slender, spiderlike fingers. His chest has a "caved-in" appearance, and he also has a modest degree of scoliosis. A midsystolic "click" is heard, and an echocardiogram reveals mitral valve prolapse. The most likely diagnosis is (A) Ehlers-Danlos syndrome. (B) Fabry disease. (C) Hurler syndrome. (D) Marfan syndrome. (E) Pompe disease.

The answer is D. Marfan syndrome, an autosomal dominant disorder caused by mutations of the fibrillin gene on chromosome 15, is a frequent cause of ectopia lentis. Other cardinal features include skeletal and cardiovascular abnormalities. Patients are tall and thin, with notably long limbs and digits. An anterior chest deformity known as pectus excavatum is sometimes seen, and vertebral abnormalities include scoliosis and lordosis. In addition, a highly arched palate and crowding of the teeth may occur. Cardiovascular complications include mitral valve prolapse and mitral regurgitation. Cystic medial necrosis can lead to dilation of the aortic root and aortic regurgitation. Life-threatening complications are aortic aneurysm and aortic dissection.

A 56-year-old woman presents with dyspnea on exertion, orthopnea, paroxysmal nocturnal dyspnea, and pulmonary edema. She also presents with severe dizziness and syncope, fatigue, weight loss, and arthralgias. After undergoing several tests, she is diagnosed with a primary heart tumor that is causing a "ball-valve obstruction" of her mitral valve. Which of the following is the most likely tumor? (A) Fibroma (B) Leiomyoma (C) Lipoma (D) Myxoma (E) Rhabdomyoma

The answer is D. Myxoma of the heart, although rare, is the most common primary cardiac tumor. Because of the jelly-like appearance and myxoid histology similar to that of some organized thrombi, the neoplastic nature of this lesion was debated for many years; however, it is now generally believed that myxoma is a true neoplasm. The most common location of myxoma is in the left atrium. Due to its location, complications may develop due to physical obstruction of blood flow through the mitral valve, resulting in symptoms of congestive heart failure. Note that while angiosarcoma is the most common primary cardiac malignancy, it is not the most common primary cardiac tumor.

A 70-year-old woman has a long history of metastatic colon cancer, and she donates her body for use in medical school anatomy courses. At death, the body is emaciated and cachectic, and gross dissection reveals small fibrin deposits arranged around the line of closure of the leaflets of the mitral valve. The valvular lesions most likely represent (A) bacterial endocarditis. (B) endocarditis of the carcinoid syndrome. (C) Libman-Sacks endocarditis. (D) nonbacterial thrombotic (marantic) endocarditis. (E) rheumatic endocarditis.

The answer is D. Nonbacterial thrombotic endocarditis, or marantic endocarditis, has been associated with a variety of wasting diseases and is observed most often in patients with cancer.

A 23-year-old woman consults an obstetrician because she is hoping to become pregnant but is concerned about possible consequences of rubella infection. She received all of her childhood immunizations, but now she has been found to be negative for antibodies to rubella. The obstetrician administers a rubella vaccination and advises her to return for an anti-rubella titer prior to becoming pregnant. Which of the following is true regarding congenital rubella infection? (A) Associated fetal defects are limited to the cardiovascular system.(B) The fetus is most vulnerable during the third trimester of pregnancy. (C) The majority of cases of congenital heart disease are caused by rubella or other intrauterine infections. (D) Patent ductus arteriosus and septal defects are the most frequent congenital cardiac abnormalities associated with rubella infection. (E) A predominant IgG antibody response indicates recent primary infection.

The answer is D. Patent ductus arteriosus and septal defects are the most frequent congenital cardiac abnormalities associated with congenital rubella infection; however, defects are not limited to the cardiovascular system, and congenital infection can also lead to deafness and mental retardation. In addition, congenital rubella infection, along with other congenital intrauterine infections, accounts for only a small proportion of cases of congenital heart disease, the majority being of unknown cause. The most severe consequences occur as a result of infection during the first trimester of pregnancy. As with all infections, an IgM antibody response indicates recent primary infection.

A 60-year-old-man is discharged after being observed in the hospital for 4 days following a myocardial infarction. He returns to his normal activities, which include sedentary work only. This point in time following a myocardial infarct is noteworthy for the special danger of which of the following? (A) Arrhythmia (B) Mural thrombosis (C) Myocardial (pump) failure (D) Myocardial rupture (E) Ventricular aneurysm

The answer is D. Rupture of the left ventricle, a catastrophic complication of acute myocardial infarction, usually occurs when the necrotic area has the least tensile strength, about 4 to 7 days after an infarction, when repair is just beginning. The anterior wall of the heart is the most frequent site of rupture, usually leading to fatal cardiac tamponade. Internal rupture of the interventricular septum or of a papillary muscle may also occur. The risk of arrhythmia is greatest within the first 6 hours after myocardial infarct. Arrhythmias are the most important early complication of acute myocardial infarction, accounting for almost 50% of deaths shortly after myocardial infarction. Myocardial, or pump, failure and mural thrombosis are other complications that may develop as a result of permanent damage to the heart after infarct. Ventricular aneurysms may develop in the fibrotic scar within 3-6 months after myocardial infarct.

A 65-year-old woman with a significant smoking history presents with cough and shortness of breath. Computed tomography of the chest reveals a central mass near the left mainstem bronchus. Biopsy of the mass is performed. Histologic examination reveals small round blue cells, and a diagnosis of small cell carcinoma is made. Which of the following is a frequent characteristic of this form of lung cancer? (A) Generally amenable to surgical cure at time of diagnosis (B) More common in women, and a less clear relation to smoking than other forms of lung cancer (C) Secretes a parathyroid-like hormone (D) Secretes either corticotrophin or antidiuretic hormone (E) Usually in a peripheral rather than in a central location

The answer is D. Small cell carcinoma of the lung is the most aggressive type of bronchogenic carcinoma. The location of this cancer is usually central. This is an undifferentiated tumor with small round blue cells and is least likely to be cured by surgery because it is usually already metastatic at diagnosis. Associated paraneoplastic syndromes include secretion of adrenocorticotropic hormone and antidiuretic hormone.

A 70-year-old retired farm worker is seen for evaluation of a pearly-appearing papule on the face just below and lateral to the left eye. The lesion is covered by small telangiectatic vessels. An excisional biopsy is performed, and the microscopic appearance is similar to that seen in the figure. Which of the following is characteristic of this disorder? (A) Distal metastases common at the time of initial diagnosis (B) Frequent origin in a preexisting actinic keratosis (C) Hamartomatous non-neoplastic lesion (D) Most frequent occurrence is on head or neck

The answer is D. The basal cell carcinoma shown in the figure has typical palisading of the nuclei of the cells at the periphery of the tumor cell clusters. Unlike squamous cell carcinoma, this tumor does not originate in preexisting actinic keratosis.

Within minutes of a bee sting, a 23-year-old woman develops generalized pruritus and hyperemia of the skin, followed shortly by swelling of the face and eyelids, dyspnea, and laryngeal edema. This reaction is mediated by (A) antigen-antibody complexes. (B) cytotoxic T cells. (C) IgA antibodies. (D) IgE antibodies. (E) IgG antibodies.

The answer is D. The clinical description is characteristic of systemic anaphylaxis, an IgE-mediated type I hypersensitivity reaction. In type I hypersensitivity, reaction of antigen with preformed IgE antibodies fixed by Fc receptors to the surface of basophils or tissue mast cells results in cytolysis and degranulation of these cells, with release of histamine and other mediators.

A 7-year-old boy presents with palpable purpura on the buttocks and legs, fever, abdominal pain and vomiting, arthritis in his knees and ankles, melena, and hematuria. His mother states that he had an upper respiratory illness approximately 1 week ago, but has otherwise been well. Blood tests reveal mild renal insufficiency. The most likely cause of the bleeding into the skin observed in this patient is (A) coagulation factor deficiency. (B) qualitative platelet dysfunction. (C) quantitative platelet dysfunction. (D) vasculitis. (E) vitamin deficiency.

The answer is D. The clinical description is that of Henoch-Schönlein purpura, a form of leukocytoclastic angiitis (hypersensitivity vasculitis) resulting from an immune reaction that damages the vascular endothelium. Henoch-Schönlein purpura is closely related to IgA nephropathy, a glomerulopathy resulting in nephritic syndrome, and may represent a systemic version of this disease.

A 40-year-old woman dies after a long history of an illness characterized by dyspnea, orthopnea, hepatomegaly, distended neck veins, and peripheral edema. The cut surface of the liver as it appears at autopsy is shown in the first panel. The second panel shows the microscopic appearance of the liver. Which of the following disorders is the most likely cause of these findings? (A) Chronic alcoholism (B) Diabetes mellitus (C) Niemann-Pick disease (D) Right-sided heart failure (e) Viral hepatitis

The answer is D. The clinical findings described in the question are typical of right-sided heart failure, as are the illustrations, which reveal the nutmeg-like appearance of hepatic chronic passive congestion. The gross morphologic appearance is caused by congested centrilobular areas alternating with pale portal areas.

A 65-year-old man who has a long history of hypertension presents to the emergency department with tearing chest pain that radiates to the back. An electrocardiogram is normal, as are cardiac enzymes. A "stat" chest radiograph demonstrates widening of the mediastinum. Which of the following is the most likely? (A) Arteriovenous fistula (B) Atherosclerotic aneurysm (C) Berry aneurysm (D) Dissecting aneurysm (E) Syphilitic aneurysm

The answer is D. The clinical presentation of a dissecting aortic aneurysm mimics that of a myocardial infarction; however, electrocardiographic changes and increased concentration of cardiac enzymes are notably absent. The mediastinum is often widened by radiographic examination. Although there is an association with hypertension and disorders of connective tissue (e.g., Marfan syndrome and Ehlers-Danlos syndrome), there is no association with atherosclerosis. The presentation exemplified by this scenario is a true surgical emergency!

A 26-year-old African-American woman has bilateral hilar adenopathy, and radiography reveals multiple reticular densities in both lung fields. A bronchoscopic biopsy reveals granulomatous inflammation with multiple giant cells of the Langhans type and no evidence of caseous necrosis. Which of the following is the most likely diagnosis? (A) Aspergillosis (B) Coccidioidomycosis (c) Histoplasmosis (d) Sarcoidosis (e) Tuberculosis

The answer is D. The histologic hallmark of sarcoidosis is the finding of noncaseating granulomatous inflammation. Although this finding is not entirely specific, a non-necrotizing granulomatous response of the lung is rarely seen in patients with tuberculosis or deep-seated fungal infections. These infections usually have a necrotizing component.

A 60-year-old man presents with feverand chills, productive cough with rusty sputum, pleuritic pain, and shortness of breathfor the past several days. A complete bloodcount reveals neutrophilia and an increasein band neutrophils. A chest radiographreveals consolidation involving the entire leftlower lobe. Which of the following microorganisms is the most likely etiologic agent? (A) Hemophilus influenzae (B) Klebsiella pneumoniae (C) Staphylococcus aureus (D) Streptococcus pneumoniae (E) Streptococcus pyogenes

The answer is D. The most common cause of lobar pneumonia is S. pneumoniae. The organism is also known as the pneumococcus, and the disease entity is often referred to as pneumococcal pneumonia.

A 35-year-old prostitute is seen in a community health care clinic. About 4 months earlier, she had a painless labial sore and swelling of a right inguinal lymph node, both of which had subsided uneventfully over a period of several weeks. About 3 weeks later she developed fever and a generalized maculopapular skin rash that involved the palms of the hands and the soles of the feet. She has developed a flattened, wart-like labial lesion that is most likely a (A) chancre (B) chancroid (C) condyloma acuminatum (D) condyloma lata (E) papillary hidradenoma.

The answer is D. The patient presents with a lesion of secondary syphilis known as condyloma lata. The original lesion, the painless ulcer or chancre, is characteristic of primary syphilis. Chancroid is a similar lesion to the primary lesion in syphilis, but instead is painful. Condyloma acuminatum is a similarly named lesion often referred to as a venereal wart and is due to HPV. Papillary hidradenoma is a common benign tumor of the vulva, presenting as a nodule that may ulcerate and bleed.

A 55-year-old woman presents with complaints of chest pain. She states that the chest pain predictably occurs when she climbs four flights of stairs to reach her apartment or when she has been jogging for more than 10 minutes. She is particularly concerned because her mother died of a myocardial infarction at 50 years of age. Which of the following best describes this patient's state? (A) Arrhythmia (B) Myocardial infarction (C) Prinzmetal angina (D) Stable angina pectoris (E) Unstable angina pectoris

The answer is D. This is a classic case of stable angina, which is chest pain that is precipitated by exertion but relieved by rest. Stable angina is due to atherosclerosis of the coronary arteries. This patient has risk factors for ischemic heart disease (IHD) (e.g., cigarette smoking, hypertension, hyperlipidemia, diabetes, family history of IHD/coronary artery disease). Prinzmetal angina is intermittent chest pain at rest, and unstable angina is prolonged chest pain at rest.

A 50-year-old female presents with restrictive lung disease. She describes an aggressive clinical course with rapidly progressive shortness of breath over the last year. A lung biopsy reveals a patchy process characterized by temporally heterogeneous areas of fibrosis. Which of the following is most likely of her expected clinical course? (A) Symptoms should abate with smoking cessation and steroid treatment (B) Prognosis is relatively good with the majority of patients surviving at 10 years (C) Excellent prognosis following removal of environmental stimulus (D) Poor prognosis with development of honeycomb lung and death within 5 years (E) Symptoms should improve with antibiotic therapy

The answer is D. This patient has UIp, which carries a dismal prognosis characterized by refractoriness to steroids and development of honeycomb lung resulting in death, often within 5 years of diagnosis. The key histologic feature is temporal heterogeneity in the fibrotic changes, whereas LIP shows chronologically uniform, diffuse fibrosis. In contrast to DIP, UIP is not related to smoking. It is not attributed to any environmental stimulus or pathogen.

A 60-year-old man presents with dyspnea on exertion and a nonproductive cough. He has never smoked, but he worked as a shipbuilder, with known asbestos exposure approximately 20 years ago. To which of the following conditions is this patient especially predisposed? (A) Acute respiratory distress syndrome (B) Goodpasture syndrome (C) Idiopathic pulmonary fibrosis (D) Idiopathic pulmonary hemosiderosis (E) Malignant mesothelioma of the pleura

The answer is E. Asbestosis is caused by inhalation of asbestos fibers, characterized by yellow-brown, rod-shaped ferruginous bodies with clubbed ends that stain positively with Prussian blue. Asbestosis results in a marked predisposition to malignant mesothelioma of the pleura or peritoneum. Exposure to asbestos is also a risk factor for primary lung carcinoma, as well as for carcinoma of the oropharynx, esophagus, and colon. The risk of primary lung carcinoma is greatly increased in cigarette smokers with exposure to asbestos.

An 8-year-old boy presents with an intensely pruritic vesicular rash and fever. Several playmates have had similar findings. The etiology of this common infectious exanthem is (A) autoimmune. (B) bacterial. (C) fungal. (D) parasitic. (E) viral.

The answer is E. Chickenpox (varicella), caused by the varicella-zoster virus, is a viral infection of childhood characterized by fever and a predominantly vesicular rash. Following overt varicella, the virus can remain latent for years in dorsal root ganglia and reappear several years later as herpes zoster (shingles).

A 21-year-old basketball player died suddenly during a game. Autopsy revealed hypertrophy of the left ventricular wall, especially of the ventricular septum. Histologically, the myocardial fibers were arranged in a disorganized pattern. Which of the following best characterizes this disorder? (A) Can be a manifestation of primary amyloidosis (B) Can be a result of myocarditis (C) Is often associated with alcohol abuse (D) Is often associated with coronary artery disease (E) Often demonstrates autosomal dominant inheritance

The answer is E. Hypertrophic cardiomyopathy, a condition that is usually inherited as an autosomal dominant disorder, is often associated with sudden death in young athletes. The ventricular septum is especially involved, with protrusion into the left ventricular cavity (asymmetric septal hypertrophy), sometimes leading to left ventricular outflow tract obstruction.

A 10-month-old girl presents with recurrent pulmonary infections, steatorrhea, and failure to thrive. Measurement of which substance is the most appropriate procedure in this patient? (A) Erythrocyte glucose-6-phosphate dehydrogenase (G6PD) (B) Serum ceruloplasmin (C) Serum β-lipoprotein (D) Serum phenylalanine (E) Sweat chloride

The answer is E. In a pediatric patient, the combination of recurrent pulmonary infections and steatorrhea (presumably due to pancreatic insufficiency) is strongly suggestive of cystic fibrosis. This disorder is characterized by a generalized defect in the reabsorption of anions, leading to increased sweat chloride concentration, an important diagnostic indicator.

A 56-year-old man dies 24 hours after the onset of substernal chest pain radiating down his left arm to the ulnar aspect of his fingertips. Which of the following morphologic myocardial findings is an indicator of irreversible injury? (A) Cell blebs (B) Depletion of glycogen (C) Mitochondrial swelling (D) Myelin figures (E) Pyknotic nuclei

The answer is E. Myelin figures, cell blebs, mitochondrial swelling, and glycogen depletion are all signs of reversible injury. Nuclear changes such as pyknosis, karyorrhexis, and karyolysis are signs of cell death and are, of course, irreversible.

A 2-year-old girl with a history of repeated pulmonary infections is found to have elevated chloride in a sweat test. An additional expected finding is (A) hypercalcemia. (B) hypotension. (C) increased metabolism. (D) renal failure. (E) steatorrhea.

The answer is E. Repeated pulmonary infections and a positive sweat test are characteristic of cystic fibrosis. In this condition, viscid secretions cause defective exocrine gland function. The lungs and pancreas are the most significant sites of involvement, and the disorder is marked by repeated bouts of pneumonia and by pancreatic failure with wasting and steatorrhea.

A 25-year-old African-American woman presents with fatigue, dyspnea, nonproductive cough, and chest pain. She does not smoke. A chest radiograph reveals prominent bilateral hilar lymphadenopathy ("potato nodules") and diffuse reticular densities in the interstitium of the lung. Laboratory studies reveal polyclonal hypergammaglobulinemia, hypercalcemia, and increased serum angiotensin-converting enzyme. Which of the following is the most likely diagnosis? (A) Acute respiratory distress syndrome (B) Adenocarcinoma of the lung (C) Eosinophilic granuloma (D) Idiopathic pulmonary fibrosis (E) Sarcoidosis

The answer is E. Sarcoidosis most often presents as a restrictive pulmonary disease that is characterized morphologically by noncaseating granulomas and can involve any organ system. Diagnostic features of note include highest incidence in persons of African lineage, somewhat higher incidence in women, bilateral interstitial pulmonary involvement, prominent hilar lymphadenopathy, polyclonal hypergammaglobulinemia, and hypercalcemia. Increased serum angiotensin-converting enzyme activity is a nonspecific indicator of granulomatous inflammation.

A 49-year-old man has a recent diagnosis of small cell carcinoma of the lung. Which of the following is an important characteristic of this form of lung cancer? (A) Ectodermal origin (B) Frequent peripheral location (C) Less association with cigarette smoking than other forms of lung cancer (D) Paraneoplastic hyperparathyroidism (E) Poorly amenable to surgery

The answer is E. Small cell carcinoma of the lung is almost always metastatic at the time of initial diagnosis and is thus poorly amenable to surgery. Despite morphologic differences, it is thought that all lung carcinomas, including small cell carcinoma, share a common endodermal origin. The location is most often central rather than peripheral, and there is a marked association with cigarette smoking. Paraneoplastic syndromes include inappropriate secretion of ACTH and ADH. Secretion of a protein with PTH-like activity is an association of squamous cell lung carcinoma.

A 30-year-old African-American woman presents with bilateral hilar lymphadenopathy and reticular densities in both lung fields. Which of the following is a defining characteristic of the disorder suggested by these findings? (A) Abnormalities restricted to lung and hilar lymph nodes (B) Hypocalcemia (C) Impaired synthesis of immunoglobulins (D) Marked hyperreactivity to tuberculin (E) Noncaseating granulomas

The answer is E. The clinical picture strongly suggests a diagnosis of sarcoidosis. The granulomas of sarcoidosis are characteristically noncaseating. Sarcoidosis is a multisystem disorder. Common findings in this highly variable disorder include anergy to tuberculin, hypercalcemia, and broad-based polyclonal hypergammaglobulinemia.

A 58-year-old man with a 700-pack-peryear smoking history presents to the emergency department with shortness of breath and hemoptysis. Portable chest radiography demonstrates a large mass centrally located within the left lung field. The serum calcium is 13.0 mg/dL (normal 8.5 to 10.2). The metabolic abnormality described here is likely due to elaboration of which substance? (A) Adrenocorticotropic hormone-like substance (B) Antidiuretic hormone (C) Carcinoembryonic antigen (D) Erythropoietin (E) Parathyroid-related hormone

The answer is E. The man is likely to have a lung tumor, given his clinical presentation and the radiographic results. The patient's hypercalcemia is likely due to a paraneoplastic syndrome, such as that due to the elaboration of parathyroid-related hormone (PTrH). PTrH is produced by squamous cell carcinoma, whereas adrenocorticotropic-like substance and antidiuretic hormone are produced by yet another form of lung cancer—small cell carcinoma of the lung. Carcinoembryonic antigen is an oncofetal antigen produced by colon cancer cells. Erythropoietin causes secondary polycythemia and is related to renal cell carcinoma.

A 55-year-old woman who died suddenly at home was found at autopsy to have suffered a rupture of the left ventricle. Which one of the following myocardial changes is the most frequent cause of this catastrophic event? (A) Abscess formation and tissue destruction due to infective endocarditis (B) Fatty change due to interaction of diphtheria exotoxin and carnitine (C) Inflammation associated with Aschoff bodies (D) Inflammation due to coxsackie B infection (E) Necrosis due to coronary artery obstruction

The answer is E. The most frequent cause of cardiac rupture is MI. This complication, which often results in hemopericardium and cardiac tamponade, occurs with peak incidence within 4 to 10 days after infarction.

A 72-year-old man presents with a 3-day history of progressively worsening productive cough, fever, chills, and signs of toxicity. Prominent physical findings include signs of consolidation and rales over the right lung base. Sputum culture is positive for Streptococcus pneumoniae.An intra-alveolar exudate filling the alveoli of the involved portion of the lung is present. Which of the following types of inflammatory cells is most likely a prominent feature of this exudate? (A) Basophils (B) Eosinophils (c) Lymphocytes (d) Monocytes-macrophages (e) Neutrophils

The answer is E. The patient has bacterial pneumonia due to Streptococcus pneumoniae, a classic example of severe acute inflammation. In the early stages of acute inflammation, the neutrophil is the most prominent inflammatory cell. It is noteworthy that, in many instances, bacterial infections are characterized by neutrophilic infiltrates. It is also noteworthy that S. pneumoniae(also known as the "pneumococcus") is the most common etiologic agent of lobar pneumonia (see Chapter 14).

An impending myocardial infarction was successfully averted by thrombolytic (clot-dissolving) therapy in a 55-year-old man. Which of the following biochemical events most likely occurred during the period of hypoxia? (A) Decreased hydrogen ion concentration (B) Increase in oxidative phosphorylation (C) Loss of intracellular Na+ and water (D) Stimulation of ATP synthesis (E) Stimulation of anaerobic glycolysis and glycogenolysis

The answer is E. The sequence of events in hypoxic cell damage is as follows: Hypoxia results in failure of oxidative phosphorylation, with resultant depletion of ATP and increase in adenosine monophosphate and adenosine diphosphate. Anaerobic glycolysis and glycogenolysis are stimulated (notinhibited) through increased phosphofructokinase and phosphorylase activities, respectively. This results in an accumulation of cell lactate, with a decrease in intracellular pH and depletion of cellular glycogen stores. Decreased availability of ATP also results in failure of the Na+K+-ATPase pump, which then leads to increased cell Na+ and water and decreased cell K+.

An HIV-positive intravenous drug user is suspected of having active tuberculosis, and a tuberculin (Mantoux) intradermal skin test is performed. After 48 hours, 10 cm of induration is observed. Which of the following are involved in this form of hypersensitivity reaction? (A) B cells and antibodies (B) Basophils and IgE (C) Immune complexes and complement (D) Plasma cells and IgM (E) T cells and macrophages

The answer is E. The tuberculin test is a classic example of delayed hypersensitivity, a form of cell-mediated hypersensitivity involving CD4+T cells and macrophages. Native CD4+T cells are converted to TH1 cells that secrete cytokines, especially interferon-γ, which is a central mediator of delayed hypersensitivity. Among the many actions of interferon-γ, the most important is the activation of macrophages.

A 10-year-old boy presents with migratory polyarthritis involving several large joints, fever, and malaise. Physical examination reveals a new heart murmur and friction rub on auscultation, and a painless nodule is detected on the extensor surface of the elbow. He had a severe sore throat approximately 2 weeks ago, apparently recovering without antibiotic therapy. The anti- streptolysin O (ASO) titer is elevated. Which of the following describes the most likely outcome for this patient? (A) Development of mitral valve stenosis over many months to years (B) Development of mitral valve stenosis over the next few months (C) Increasing severity of the current symptoms and findings over the next few decades (D) Persistence of the current symptoms and signs over the patient's lifetime (E) Total recovery after 1 to 2 months with no further complications or sequelae

The answer is E. This is a case of acute rheumatic fever. Acute rheumatic fever manifests most commonly in patients 5 to 15 years of age with migratory polyarthritis, pancarditis, subcutaneous nodules, erythema marginatum, and Sydenham chorea. Decades later, severe valvular disease, often manifesting as mitral stenosis, may develop as a feature of rheumatic heart disease. In this chronic stage of rheumatic disease, fibrotic valves may become stenotic, insufficient, or both, but much more commonly, progression to cardiac valve complication does not occur.

A 3-year-old girl presents to the emergency department with fever, hoarseness, a "seal bark-like" cough, and inspiratory stridor. Her father states that she has had a cold for the past few days, with runny nose, nasal congestion, sore throat, and cough. He is now concerned because her cough has become loud, harsh, and brassy. Which of the following is the most likely cause of her ailment? (A) Fungus (B) Gram-negative bacteria (C) Gram-positive bacteria (D) Parasite (E) Virus

The answer is E. This is a classic case of acute laryngotracheobronchitis (croup), an acute inflammation of the larynx, trachea, and epiglottis. The most common cause of croup is a viral (parainfluenza virus type I) infection.

A 45-year-old woman with no smoking history presents with new onset of cough, shortness of breath, and weight loss. Imaging reveals a peripherally located lung mass with no evidence of primary tumor elsewhere in the body. On light microscopy, the tumor is comprised of poorly formed glands. Which of the following is most likely to be true of this tumor? (A) Positive for synaptophysin immunostaining (B) Associated squamous dysplasia at the periphery of the tumor (C) Most likely already metastatic, therefore surgery is not recommended (D) Driven by tobacco-associated carcinogens (E) Positive for mutation in EGFR

The answer is E. This woman has an adenocarcinoma. These are the most likely lung cancers to arise in never-smokers and are more common in women. They have been associated with EGFRmutations, and EGFRmutation testing is rapidly becoming the standard-of-care for lung adenocarcinomas in order to direct chemotherapy (EGFRmutated cases may respond to targeted tyrosine kinase inhibitors). Adenocarcinomas may be preceded by or associated with atypical adenomatous hyperplasia, not squamous dysplasia. Unlike small cell carcinomas, they do not show neuroendocrine features such as synaptophysin staining and metastasis at the time of presentation is not the rule.

A 56-year-old man is surgically treated by a four-vessel coronary artery bypass graft procedure and placed on prophylactic daily aspirin therapy. Aspirin has been shown to prevent recurrent myocardial infarction through its ability to inhibit the synthesis of (A) adenosine diphosphate (ADP). (B) leukotriene B4(LTB4). (C) nitric oxide (NO). (D) prostaglandin I2(PGI2). (e) thromboxane A2(TxA2).

The answer is E. Thromboxane A2(TxA2) promotes platelet aggregation, as does ADP. Aspirin irreversibly inhibits the enzymes cyclooxygenase 1 and 2 and thereby the synthesis of TxA2, thus inhibiting platelet aggregation, which is thought to be an important early step in atherogenesis. A negative but apparently unimportant consequence of aspirin prophylaxis is the parallel inhibition of synthesis of the antiaggregant endothelial PGI2, also a product of the cyclooxygenase pathway.

A 46-year-old woman presents with fever, hemoptysis, weight loss, and night sweats. She has never smoked. She recently returned from a month-long trip to Asia. A chest radiograph reveals apical lesions with cavitation in the left lung. A purified protein derivative (PPD) test is placed, and 48 hours later an 18-mm wheal develops. Sputum cultures reveal numerous acid-fast organisms. This patient is put on contact precautions, and a regimen for tuberculosis is started. Which of the following disorders does this patient most likely have? (A) Acquired immunodeficiency syndrome (B) Congenital immunodeficiency (C) Miliary tuberculosis, with seeding of distal organs with innumerable small millet seed-like lesions (D) Primary tuberculosis, characterized by the Ghon complex (E) Secondary tuberculosis, resulting from activation of a prior Ghon complex, with spread to a new pulmonary site

The answer is E. Tuberculosis, at one time a frequent hazard in the United States, is now relatively uncommon except in immunocompromised individuals and persons returning from parts of the world where the disease remains a common problem. Primary tubercu-losis is the initial infection by M. tuberculosis, and is restricted to the primary, or Ghon, complex, a combination of a peripheral subpleural parenchymal lesion and involved hilar lymph nodes. Cavitation and selective localization to the pulmonary apices are characteristics of secondary tuberculosis. Secondary tuberculosis may spread through the lymphatics and blood to other organs, resulting in miliary tuberculosis.


संबंधित स्टडी सेट्स

Nursing Application-Assessment and Care

View Set

Delivering Project Benefits and Value

View Set

XCEL Chapter 2: Legal Concepts of Insurance Contract

View Set